Find the range of the data set. Round to the nearest tenth.​

Answers

Answer 1

Range = Highest - Lowest

Range = 23,950 - 6,950

Range = 17,000

Must click thanks and mark brainliest

Answer 2

Answer:

The range is 17000.

Step-by-step explanation:

How you find range is by finding the difference between the maximum and minimum number from the data set.

So, the maximum number is 23950 and minimum is 6950 from the data set.

23950 - 6950 = 17000.


Related Questions

Find f(-3) for f(x) = 4(2)^x
O A. -32

O B. 1/2

O C. -24

O D. 1/8

Answers

Answer:

B. 1/2

General Formulas and Concepts:

Pre-Algebra

Order of Operations: BPEMDAS

Brackets Parenthesis Exponents Multiplication Division Addition Subtraction Left to Right

Algebra I

Functions

Function Notation

Step-by-step explanation:

Step 1: Define

Identify

f(x) = 4(2)ˣ

Step 2: Evaluate

Substitute in x [Function f(x)]:                                                                           f(-3) = 4(2)⁻³Exponents:                                                                                                        f(-3) = 4(1/8)Multiply:                                                                                                             f(-3) = 1/2

what is 3x10x178 I need help asap

Answers

Answer:

3x10=30   30x178=5,430

Step-by-step explanation:

QUESTION:- what is 3x10x178

ANSWER:-

[tex]3 \times 10 \times 178 \\ 30 \times 178 \\ 5340 \: \: answer[/tex]

Can someone help me with this math homework please!

Answers

Answer:

f(n+1) = f(n) - 5

Step-by-step explanation:

Just find some relationship between 2 numbers that are next to each other.

which of the following pairs of equations are perpendicular

Answers

Answer:

were are the pairs?

Which table of values could be generated by the equation 10x+5y=15? (Will give brainlest and 21 points)

Answers

Answer:

(For the image) A

Step-by-step explanation:

 The angle of elevation to a nearby tree from a point on the ground is measured to be 65°. How tall is the tree if the point on the ground is 92 feet from the tree? Round your answer to the nearest hundredth of a
foot if necessary.

Answers

Answer:

197.3 feet

Step-by-step explanation:

197.295 rounded to the nearest hundredth is 197.30 or 197.3

The height of the tree if the point on the ground from the tree is 92 feet will be 197.29 feet.

What is a right-angle triangle?

It is a type of triangle in which one angle is 90 degrees and it follows the Pythagoras theorem and we can use the trigonometry function. The Pythagoras is the sum of the square of two sides is equal to the square of the longest side.

The angle of elevation to a nearby tree from a point on the ground is measured to be 65°.

The height of the tree if the point on the ground from the tree is 92 feet. Then we have

Let h be the height of the tree. Then we have

[tex]\tan 65^o = \dfrac{h}{92}\\\\ 2.1445\ = \dfrac{h}{92}[/tex]

Then we have

[tex]\rm h = 2.1445 \times 92\\\\h = 197.29\ ft[/tex]

More about the right-angle triangle link is given below.

https://brainly.com/question/3770177

A car travels 600 km in 6 hours. at what rate of speed is the car traveling?

Answers

Step-by-step explanation:

100Km/hours......V/T

Find the length of side BC give your answer to three significant figures

Answers

Answer:

19.4 cm

Step-by-step explanation:

Hi there!

This is a right triangle. We're given an angle, the side adjacent to the angle and we're solving for the hypotenuse. Given this information, we can use the cosine ratio:

[tex]cos\theta=\frac{adj}{hyp}[/tex]

Plug in the given angle and side

[tex]cos71=\frac{6.3}{BC}\\BC=\frac{6.3}{cos71} \\BC=19.4[/tex]

Therefore, the length of BC is 19.4 cm when rounded to 3 significant figures.

I hope this helps!

What is the range of f(x)=4^x

Answers

Answer:

B

Step-by-step explanation:

At - infinity, the function will tend to 0 and at +infinity, the function will tend to +infinity. Those are the two extremas of the function and extremas define the range. Range is all positive real number

Find the measure of the missing angle using the exterior angle sum theorm.

Answers

Answer:

95 degrees

Step-by-step explanation:

First find the angles within the triangle. 180 - 43 - 52 = 85, so the missing angle within the triangle is 85. The exterior angle would be supplementary to that, so 180 - 85 = 95.

Aisha wants to paint the four walls of her living room.
Each wall is 2.2 m high and 5.5 m long.
One wall has a door of 1.8 m by 0.9 m.
Tins of paint cost £13 per 2 L tin.
Each litre of paint can cover 8 m2 of wall.
There is an offer of: Buy 2 tins get the 3rd at half price.
How much will Aisha pay to paint her living room?

Answers

Answer:

£32.50

Step-by-step explanation:

my first question to the teacher : so, no windows in the living room ?

so, it is a square living room with 5.5 m side length.

but each wall is a rectangle of 2.2 × 5.5 m.

for one wall we have to deduct a door area of 1.8×0.9 m.

so, one wall

2.2 × 5.5 = 12.1 m²

4 walls

4 × 12.1 = 48.4 m²

minus one door area

1.8 × 0.9 = 1.62 m²

48.4 - 1.62 = 46.78 m² total paint area

1 L paint covers 8 m².

so, we need 46.78/8 = 5.85 liters.

she gets the paint in 2 L tins. so, she needs 3 tins (6 L).

each tin costs £13.

and because she buys 3 tins, she gets the third one for half the price (13/2 = £6.50).

so, she has to pay

2×13 + 6.50 = 26 + 6.50 = £32.50


use a double angle or half angle identity to find the exact value of each expression

Answers

Answer:

Step-by-step explanation:

There are 2 very distinct and important things that we need to know before completing the problem. First is that we are given that the cos of an angle is 1/3 (adjacent/hypotenuse) and it is in the first quadrant. We also need to know that the identity for sin2θ = 2sinθcosθ.

We already know cos θ = 1/3, so we need now find the sin θ. The sin ratio is the side opposite the angle over the hypotenuse, and the side we are missing is the side opposite the angle (we do not need to know the angle; it's irrelevant). Set up a right triangle in the first quadrant and label the base with a 1 (because the base is the side adjacent to the angle), and the hypotenuse with a 3. Find the third side using Pythagorean's Theorem:

[tex]3^2=1^2+y^2[/tex] which simplifies to

[tex]9=1+y^2[/tex] and

[tex]y^2=8[/tex] so

[tex]y=\sqrt{8}=2\sqrt{2}[/tex] so that's the missing side. Now we can easily determine that

[tex]sin\theta=\frac{2\sqrt{2} }{3}[/tex]

Now we have everything we need to fill in the identity for sin2θ:

[tex]2sin\theta cos\theta=2(\frac{2\sqrt{2} }{3})(\frac{1}{3})[/tex] and multiply all of that together to get

[tex]2sin\theta cos\theta=\frac{4\sqrt{2} }{9}[/tex]

please help me its urgent​

Answers

Answer:

number of students who like only iphone is 36...

Step-by-step explanation:

100 is the total number..

from that 40 like both so we subtract..

100-40=60..

60 = 2x + 3x..

60 / 5 = 12..

so numbet who like iphone is 3x which is 3 * 12=36..

Grandma is making a quilt. She has 540 cm of fabric to border the quilt. What is the greatest possible area for the quilt?
Question 1 options:


11 664 cm^2


18225 cm^2


72900 cm^2


291600 cm^2

Show your work:

Answers

Answer:

18225 cm²

Step-by-step explanation:

Divide 540 by 4 to get the length of all sides

540/4 = 135

Square 135 to get the max possible size

135² = 18225

18225 cm²  is the greatest possible area for the quilt.

What is area?

The measurement that expresses the size of a region on a plane or curved surface is called area. Surface area refers to the area of an open surface or the boundary of a three-dimensional object, whereas the area of a plane region or plane area refers to the area of a form or planar lamina.

Given

Divide 540 by 4 to obtain the length of all sides

540/4 = 135

Square 135 to acquire the max possible size

135² = 18225

18225 cm²  is the greatest possible area for the quilt.        

To learn more about area refer to:

https://brainly.com/question/25292087

#SPJ2

Three red balls, 5 green balls and a number of blue balls are put together in a sac. One ball is picked at random from the sac. If the probability of picking a red ball is 1|6, find the a) The number of blue balls in sac. B) the probability of picking a green ball​

Answers

Answer:

total balls = 18 .... 3/x = 1/6

blue = 10 ... 18-(5+3) = 10

p of green = 5/18 = .277

Step-by-step explanation:

Convert to decimal degrees.

-(167° 31”)

[?]°

Enter your answer with three decimal places.

Answers

Answer:

The angle in decimal form is 167.009°.

Step-by-step explanation:

We know an angle in terms of integer angles, minutes and seconds, whose conversion into decimal degrees is expressed by the following formula:

[tex]\theta = n + \frac{m}{60}+\frac{s}{3600}[/tex] (1)

Donde:

[tex]n[/tex] - Integer angle, in sexagesimal degrees.

[tex]m[/tex] - Minutes.

[tex]s[/tex] - Seconds.

If we know that [tex]n = 167[/tex], [tex]m = 0[/tex] and [tex]s = 31''[/tex], then the angle in decimal form is:

[tex]\theta = 167^{\circ}+\frac{0}{60}^{\circ} + \frac{31}{3600}^{\circ}[/tex]

[tex]\theta = 167.009^{\circ}[/tex]

The angle in decimal form is 167.009°.

Find the length of the segment indicated. Round your answer to the nearest 10th if necessary.

Answers

Answer:

x=13.6

Step-by-step explanation:

By Pythagoras theorem, 5.5^2+x^2=14.7^2. x^2=14.7^2-5.5^2. x=13.6

Find cosθ+cos3θ+cos5θ+cos7θ by using the Sum-to-Product Formula.
Please also show your work as well. Thanks!

Answers

Answer:

[tex] \rm\displaystyle 4\cos( \theta) \cos \left( {2\theta} \right) \cos \left( {4 \theta } \right) [/tex]

Step-by-step explanation:

I assume the question want us to rewrite cosθ+cos3θ+cos5θ+cos7θ by using Sum-to-Product Formula and note that it's not an equation therefore θ can never be specified

===========================

so we want to rewrite cosθ+cos3θ+cos5θ+cos7θ by using Sum-to-Product Formula the good news is that the number of the function of the given expression is even so there's a way to do so, rewrite the expression in parentheses notation:

[tex] \rm\displaystyle \left( \cos( \theta) + \cos(3 \theta) \right) + \left(\cos(5 \theta) + \cos(7 \theta) \right)[/tex]

recall that,Sum-to-Product Formula of cos function:

[tex] \rm \boxed{\displaystyle \cos( \alpha ) + \cos( \beta ) = 2 \cos \left( \frac{ \alpha + \beta }{2} \right) \cos \left( \frac{ \alpha - \beta }{2} \right) }[/tex]

notice that we have two pair of function with which we can apply the formula thus do so,

[tex] \rm\displaystyle \left( 2\cos \left( \frac{ \theta + 3 \theta}{2} \right)\cos \left( \frac{ \theta - 3 \theta}{2} \right) \right) + \left(2\cos \left( \frac{5 \theta + 7 \theta}{2} \right) \cos \left( \frac{5 \theta - 7 \theta}{2} \right) \right)[/tex]

simplify addition:

[tex] \rm\displaystyle \left( 2\cos \left( \frac{4 \theta}{2} \right)\cos \left( \frac{ - 2\theta }{2} \right) \right) + \left(2\cos \left( \frac{12 \theta }{2} \right) \cos \left( \frac{ - 2 \theta}{2} \right) \right)[/tex]

simplify division:

[tex] \rm\displaystyle \left( 2\cos \left( {2 \theta} \right)\cos \left( { - \theta } \right) \right) + \left(2\cos \left( {6 \theta } \right) \cos \left( { - \theta} \right) \right)[/tex]

By Opposite Angle Identities we acquire:

[tex] \rm\displaystyle \left( 2\cos \left( {2 \theta} \right)\cos \left( { \theta } \right) \right) + \left(2\cos \left( {6 \theta } \right) \cos \left( { \theta} \right) \right)[/tex]

factor out 2cosθ:

[tex] \rm\displaystyle 2 \cos( \theta) (\cos \left( {2 \theta} \right) + \cos \left( {6 \theta } \right) )[/tex]

once again apply Sum-to-Product Formula which yields:

[tex] \rm\displaystyle 2 \cos( \theta) (2\cos \left( {4\theta} \right) \cos \left( {2 \theta } \right) )[/tex]

distribute:

[tex] \rm\displaystyle 4\cos( \theta) \cos \left( {2\theta} \right) \cos \left( {4 \theta } \right) [/tex]

and we're done!

Find cos 0
A. 15/8
B. 15/17
C. 8/15
D. 8/17

Answers

Answer:

A.15/8

Step-by-step explanation:

the answer is 15/8

Answer:

D.

[tex]{ \tt{ \cos( \theta) = \frac{adjacent}{hypotenuse} }} \\ \\ { \tt{ \cos( \theta) = \frac{8}{ \sqrt{ {15}^{2} + {8}^{2} } } }} \\ \\ { \tt{ \cos( \theta) = \frac{8}{ \sqrt{289} } }} \\ \\ { \tt{ \cos( \theta) = \frac{8}{17} }}[/tex]

If f is continuous for all x, which of the following integrals necessarily have the same value?

Answers

Answer:

B

Step-by-step explanation:

Given the three integrals, we want to determine which integrals necessarily have the same value.

We can let the first integral be itself.

For the second integral, we can perform a u-substitution. Let u = x + a. Then:

[tex]\displaystyle du = dx[/tex]

Changing our limits of integration:

[tex]u_1=(0)+a=a \text{ and } u_2 = (b+a)+a = b+2a[/tex]

Thus, the second integral becomes:

[tex]\displaystyle \int_{0}^{b+a}f(x+a)\, dx = \int_a^{b+2a} f(u)\, du[/tex]

For the third integral, we can also perform a u-substitution. Let u = x + c. Then:

[tex]\displaystyle du = dx[/tex]

And changing our limits of integration:

[tex]\displaystyle u_1=(a-c)+c=a \text{ and } u_2=(b-c)+c=b[/tex]

Thus, our third integral becomes:

[tex]\displaystyle \int_{a-c}^{b-c}f(x+c)\, dx = \int_{a}^{b} f(u)\, du[/tex]

Since the only difference between f(x) and f(u) is the variable and both the first and third integral have the same limits of integration, our answer is B.

A square has a side length of 36 feet. This square is dilated by a scale factor of 2/3 to create a new square. What is the side length of the new square?

Answers

Answer:

24

Step-by-step explanation:

Multiply the side length by the dilation

36 x 2/3

72/3

Simplify

72/3 = 24

Your answer is correct

If the speed of an object in motion is doubled, its kinetic energy becomes how many times the original kinetic energy

Answers

Answer: Becomes four times

Step-by-step explanation:

Given

Speed is doubled for a moving object

Suppose initial speed is u

Increased speed is 2u

Kinetic Energy is given by

[tex]\Rightarrow K=0.5mu^2[/tex]

When speed is doubled

[tex]\Rightarrow K'=0.5m(2u)^2\\\Rightarrow K'=(0.5mu^2)\times 4\\\Rightarrow K'=4K[/tex]

Kinetic energy becomes four times

cos theta / sec theta -1 - sin theta / 1+cos theta = 2 cot theta​

Answers

Step-by-step explanation:

Explanation is in the attachment

Hope it is helpful to you

An angle measures 73.6° less than the measure of its supplementary angle. What is the measure of each angle?

Answers

Answer:

Smaller angle = 53.2

Larger angle = 126.8

Step-by-step explanation:

Lets say x is the measure of the supplement. Since we know they're supplementary, we know their angle measure sum will equal 180. We can set up our equation like this [tex]x + (x-73.6) = 180[/tex]. Note: (x - 73.6) is the measure of the smaller angle. By solving, we get 126.8 degrees for the measure of the supplement. If we plug in the value of x into (x-73.6), we get 53.2 degrees as the angle measure of the smaller angle.

Please help! Identify an equation in point-slope form for the line parallel to y=3/4x-4 that passes through (-1,7).

Answers

The answer is c y-7=3/4(x+1) hope this helps.

can someone please help

Answers

Answer:

see image

Step-by-step explanation:

State what additional information is required in order to know that the triangle in the image below are congruent for the reason given…

Reason: HL Postulate

Answers

Answer:

FG ≈ FL (Both are hypotenuse, supposed to be equal in order to the congruency to become HL)

Answered by GAUTHMATH

A house on the market was valued at $472,000. After several years, the value increased by 19%. By how much did the house's value increase in dollars? What is the current value of the house?

Answers

Step-by-step explanation:

Increase in dollars

19/100 x 472.000 = $89,670

and the current value house is $472,000 + $89,670 = $561,680

The difference between 15 and 9 is subtracted from 5 times the sum of 7 and 3​

Answers

Answer:

44

Step-by-step explanation:

The difference between 15 and 9 is 6. 5 times the sum of 7 and 3 is 50 because 7+3=10 and 10 times 5 is 50. So if you subtract the difference between 15 and 9 from 50 you get 44.

6. Solve the triangle by finding the length of DF and the measures of all the angles. For side lengths, round to the
nearest tenth. For angles, round to the nearest degree. (2 points)
D
5 ft
E
DF =
D
LE=
5.83 ft
ZF =
F

Answers

Answer:

∠F = 59°

∠E = 31°

DF = ≅ 3  (2.98)

Step-by-step explanation:

Other Questions
Which one of the following compoundscontains ionic bonds?a) NI3B) HFc) SiO2d) CaO The change that occurs in 2nd and3rd person in the following verb is aussehen - du siehst - er/sieles siehtA "ehr changes to st"B. "ehr changes to iehC. "ehr changes to "T If a line has a midpoint at (2,5), and the endpoints are (0,0) and (4,y), what is the value of y? Please explain each step for a better understanding:) write your answer in simplest radical form El peso mximo de una carga, recomendado por el INSST, para su manipulacin sin riesgo dorsolumbar es de result of 5 and 75 with dividid by 3 What are the three functions of carbohydrates?(For class 7) Your class will like to visit an important and interesting place.write to the one in charge of the place and give three reason why you want to visit the place what does this equal 2^3 + 6^5= Determine three consecutive odd integers whose sum is 2097. What are direct quotes from "King and the Dragonflies" that tie into the theme of the story? PLEASE HELP I WILL GIVE BRAINLIEST in what condition does a body float please hlep me its ixl Every man dies. Not every man really lives.William Wallace Place the following elements in order of decreasing atomic size: lead, phosphorus, oxygen, cesium, barium, and silicon. Rank from largest to smallest. To rank items as equivalent, overlap them.Cs, O, P, Si, Ba, Pb from the21. Space-produced photographs showed that air pollution..industrial Belts in the middle latitudes.A. is being exportedB. exportedC. has been exportedD. will exportHelp me! Rationalise the denominator What are the level wise professions? A 3-gallon jug of water costs $15.00. What is the price per quart?